LSAT and Law School Admissions Forum

Get expert LSAT preparation and law school admissions advice from PowerScore Test Preparation.

User avatar
 Dave Killoran
PowerScore Staff
  • PowerScore Staff
  • Posts: 5862
  • Joined: Mar 25, 2011
|
#41581
Complete Question Explanation
(The complete setup for this game can be found here: lsat/viewtopic.php?t=11733)

The correct answer choice is (A)

The line 2 ticket that must be G is the February ticket. Because no other February ticket can be G, from the fifth rule we know that the January line 3 ticket is G. This forces the January line 1 ticket to be R, and from that inference we can deduce that the February line 1 ticket is not R, and instead must be Y. Because one February ticket must be R, we can deduce that February line 3 is R, leading to the following setup:
J91_Game_#4_#20_diagram 1.png
Hence, answer choice (A) is correct.
You do not have the required permissions to view the files attached to this post.
 sajanlama@gmail.com
  • Posts: 3
  • Joined: Jul 05, 2022
|
#97658
I have spent over an hour trying to figure why this answer for this question is not A, C, or E. I do not see it, perhaps its the late night hours getting to me. I would still appreciate a second eye on this please.

The key issue I am able to deduce to is that I cannot see is why "Exactly two of the six sets of tickets must be red"

Thank you for your help. :-?
 sajanlama@gmail.com
  • Posts: 3
  • Joined: Jul 05, 2022
|
#97659
Sorry let me rephrase. I believe all of the answer choices of this question is wrong because B and D are must be false. A, C, and E are could be true so it fails to satisfy the question of must be true.
 sajanlama@gmail.com
  • Posts: 3
  • Joined: Jul 05, 2022
|
#97675
Never mind everyone! I was able to see it after some sleep.

Get the most out of your LSAT Prep Plus subscription.

Analyze and track your performance with our Testing and Analytics Package.